Difference between revisions of "1983 AHSME Problems/Problem 26"
Sevenoptimus (talk | contribs) (Added a solution) |
Sevenoptimus (talk | contribs) m (Improved readability of solution) |
||
Line 12: | Line 12: | ||
==Solution== | ==Solution== | ||
− | Firstly note that <math>p \leq \frac{3}{4}</math> and <math>p \leq \frac{2}{3}</math>, as clearly the probability that both <math>A</math> and <math>B</math> occur cannot be more than the probability that <math>A</math> or <math>B</math> alone occurs. The more restrictive condition is <math>p \leq \frac{2}{3}</math>, since <math>\frac{2}{3} < \frac{3}{4}</math>. Now, by the Inclusion-Exclusion Principle, we also have < | + | Firstly note that <math>p \leq \frac{3}{4}</math> and <math>p \leq \frac{2}{3}</math>, as clearly the probability that both <math>A</math> and <math>B</math> occur cannot be more than the probability that <math>A</math> or <math>B</math> alone occurs. The more restrictive condition is <math>p \leq \frac{2}{3}</math>, since <math>\frac{2}{3} < \frac{3}{4}</math>. |
+ | |||
+ | Now, by the Inclusion-Exclusion Principle, we also have <cmath>\text{P}(A' \wedge B') = 1 - \text{P}(A) - \text{P}(B) + \text{P}(A \wedge B) = 1 - \frac{3}{4} - \frac{2}{3} + p = p - \frac{5}{12},</cmath> and as a probability must be non-negative, <math>p - \frac{5}{12} \geq 0</math>, so <math>p \geq \frac{5}{12}</math>. Therefore, combining our inequalities gives <math>\frac{5}{12} \leq p \leq \frac{2}{3}</math>, which is answer choice <math>\boxed{\textbf{D}}</math>. |
Revision as of 17:36, 27 January 2019
Problem
The probability that event occurs is ; the probability that event B occurs is . Let be the probability that both and occur. The smallest interval necessarily containing is the interval
Solution
Firstly note that and , as clearly the probability that both and occur cannot be more than the probability that or alone occurs. The more restrictive condition is , since .
Now, by the Inclusion-Exclusion Principle, we also have and as a probability must be non-negative, , so . Therefore, combining our inequalities gives , which is answer choice .